Last visit was: 24 Apr 2024, 20:30 It is currently 24 Apr 2024, 20:30

Close
GMAT Club Daily Prep
Thank you for using the timer - this advanced tool can estimate your performance and suggest more practice questions. We have subscribed you to Daily Prep Questions via email.

Customized
for You

we will pick new questions that match your level based on your Timer History

Track
Your Progress

every week, we’ll send you an estimated GMAT score based on your performance

Practice
Pays

we will pick new questions that match your level based on your Timer History
Not interested in getting valuable practice questions and articles delivered to your email? No problem, unsubscribe here.
Close
Request Expert Reply
Confirm Cancel
SORT BY:
Date
Tags:
Difficulty: 555-605 Levelx   Complete the Passagex                  
Show Tags
Hide Tags
avatar
Intern
Intern
Joined: 11 Jul 2012
Posts: 45
Own Kudos [?]: 6618 [96]
Given Kudos: 0
Send PM
Most Helpful Reply
User avatar
Manhattan Prep Instructor
Joined: 30 Apr 2012
Posts: 782
Own Kudos [?]: 2583 [33]
Given Kudos: 5
Send PM
Senior Manager
Senior Manager
Joined: 17 Mar 2010
Status:Final Countdown
Posts: 320
Own Kudos [?]: 1305 [12]
Given Kudos: 76
Location: United States (NY)
GPA: 3.82
WE:Account Management (Retail Banking)
Send PM
Tutor
Joined: 16 Oct 2010
Posts: 14817
Own Kudos [?]: 64904 [3]
Given Kudos: 426
Location: Pune, India
Send PM
Re: A recent government study links the high rates of respiratory ailments [#permalink]
2
Kudos
1
Bookmarks
Expert Reply
betterscore wrote:
A recent government study links the high rates of respiratory ailments in Groverston to airborne pollutants released by the Woodco plywood manufacturing plant there. To address the problem, the government imposed strict regulations on emissions which will go into effect in four years. Although Woodco plans to cut its emissions in half two years ahead of schedule, it is unlikely that the
rate of respiratory ailments will decline before the regulations go into effect, since _____________ .

Which of the following most logically completes the passage?

(A) the number of facilities capable of treating respiratory ailments is not likely to increase

(B) reducing emissions even further than planned would necessitate decreasing production at Woodco

(C) it is difficult to make accurate, long-term predictions about emissions

(D) not all respiratory ailments are caused by airborne pollutants

(E) three new plywood manufacturing plants are about to go into production in Groverston


Respiratory ailments linked to airborne pollutants released by the Woodco plywood manufacturing plant.
Government imposed strict regulations on emissions (on all emissions, not just of Woodco) which will go into effect in four years.
Woodco plans to cut its emissions in half two years ahead of schedule
But rate of respiratory ailments will not decline before the regulations go into effect because ...

So the argument tells us this - The ailments are linked to Woodco's plywood plant. So the govt has imposed strict reg on emissions. Though Woodco will reduce emission sooner, but rate of ailments is not likely to reduce before regulations come into effect. Why? Why will the ailments not reduce before regulations even though Woodco will reduce emissions?
(e) three new plywood manufacturing plants are about to go into production in Groverston
Woodco is a plywood manufacturing plant. Very likely that any plywood manufacturing plant will have similar emissions. So if new plants are going into production soon, Woodco reducing its emissions won't help much till the time the regulations come into effect. The "strict" regulations are for all so they will help reduce emissions and hence reduce ailments. Correct.

Answer (E)

Note that all other options are completely irrelevant.
If you evaluate the argument properly, you need to spend little time on options.
General Discussion
User avatar
Manager
Manager
Joined: 19 Oct 2011
Posts: 87
Own Kudos [?]: 1235 [3]
Given Kudos: 33
Location: India
Send PM
Re: A recent government study links the high rates of respiratory ailments [#permalink]
3
Kudos
In the logically complete the passage questions/must be true question, we should stick to the entities given in the passage and the AO should pass the FACT TEST .
One can easily narrow it down to D/E.
D is incorrect because it broadens the scope of discussion.
E is correct.
User avatar
Current Student
Joined: 15 Sep 2012
Status:Done with formalities.. and back..
Posts: 525
Own Kudos [?]: 1187 [1]
Given Kudos: 23
Location: India
Concentration: Strategy, General Management
Schools: Olin - Wash U - Class of 2015
WE:Information Technology (Computer Software)
Send PM
Re: A recent government study links the high rates of respiratory ailments [#permalink]
1
Kudos
tingting85114 wrote:
Which of the following most logically completes the passage below?
Heavy rains during Centralia’s corn planting season prevented some farmers there from planting corn. It is now the planting season for soybeans, another of Centralia’s principal crops, and those fields originally intended for corn are dry enough for planting. Nonetheless, even though soybean prices are unusually high at present, the farmers will leave most of these fields empty rather than plant them with soybeans, since ‗‗‗‗‗‗.

A. the extensive rains have led to an increase in the price of corn
B. some Centralian farmers anticipate serious financial losses due to the extremely wet spring planting season
C. chemicals that were used to prepare the fields for corn planting would stunt the growth of soybeans
D. the majority of Centralia’s corn farmers were able to plant corn as they had intended, despite the wet planting season
E. many Centralian farmers grow both corn and soybeans


Basically, we are looking for the reason for the discrepency highlighted by "even though soybean prices are unusually high at present, the farmers will leave most of these fields empty rather than plant them with soybeans"


A. the extensive rains have led to an increase in the price of corn
The planting season for corn is over. price increase must be due to limited supply. It is unlikely that increase in price can lead farmers to plant corn in a non season. Hence not the reason.

B. some Centralian farmers anticipate serious financial losses due to the extremely wet spring planting season
irrelevant... We dont know which season is.. spring planting season? we know about corn planting season and soybean planting seasons. Not the reason.

C. chemicals that were used to prepare the fields for corn planting would stunt the growth of soybeans
If the growth of soybean is hampered due to chemicals, farmers would again suffer a loss. Hence they might not plant the fields with soybeans. Possible reason.

D. the majority of Centralia’s corn farmers were able to plant corn as they had intended, despite the wet planting season
Totally irrelevant. Farmers were able to plant corn as they intended. But it doesnt help in explaining the discrepancy. Not the reason.

E. many Centralian farmers grow both corn and soybeans
Again ,irrelavant. Yeah we know that - both are principal crops in centralia. But it doesnt help in explaining the discrepancy.Not the reason.

C presents only possible reason.

Hence Ans C.
avatar
Intern
Intern
Joined: 07 Oct 2012
Posts: 6
Own Kudos [?]: [0]
Given Kudos: 1
Send PM
Re: A recent government study links the high rates of respiratory ailments [#permalink]
OG – 13. Q # 59

Which of the following most logically completes the passage?

A recent government study links the high rates of respiratory ailments in Groverston to airborne pollutants released by the Woodco plywood manufacturing plant there. To address the problem the government imposed strict regulations on emissions which will go into effect in four years. Although Woodco plans to cut its emissions in half two years ahead of schedule, it is unlikely that the rate of respiratory ailments will decline before the regulations go into effect, since _______.

A.the number of facilities capable of treating respiratory ailments is not likely to increase
B.reducing emissions even further than planned would necessitate decreasing production at Woodco
C. it is difficult to make accurate, long-term predictions about emissions
D.not all respiratory ailments are caused by airborne pollutants
E.three new plywood manufacturing plants are about to go into production in Groverston

OA: E

Why is D not the valid answer? There can be other factors as well which leads to respiratory ailments and these factors will only decline after the government regulations go into effect.

Pls help clarify this doubt. I was very confident of Option ‘D’. Was actually surprised to see that this is not the OA


OG – 13, Q # 65


Which of the following logically completes the passage?

Pecan growers get a high price for their crop when pecans are comparatively scarce, but the prices drops sharply when pecans are abundant.
Thus, in high yield years, growers often hold back part of their crop in refrigerated warehouses for one or two years, hoping for higher prices in future.This year's pecan crop was the smallest in five years. It is nonetheless quite possible that a portion of this year's crop will be held back,since

A) Each of the last two years produced record breaking pecan yields
B) The Quality of this year's pecan crop is no more worse than the quality of the pecan crops of the previous five years
C) pecan prices have not been to sharp fluctuation in recent years
D) For some pecan growers , this year's crop was no smaller than last year's.
E) the practice of olding back part one year's crop had not yet become widespread the last time the pecan crop was as small as it was this year.

OA:A

Why is ‘E’ wrong?

Option E states that the practice of holding back the crops had not become widespread which means that the crops were sold which means that the pecan growers had no inventory.

Furthermore, the quantity was the same. Hence they will hold back the crops this year in order to gain more profits in future



OG – 13 . Q # 67

Several industries have recently switched at least partly from older technologies powered by fossil fuels to new technologies powered by electricity. It is thus evident that less fossil fuel is being used as a result of the operations of these industries than would have been used if these industries had retained their older technologies.


Which of the following, if true, most strengthens the argument above?


(A) Many of the industries that have switched at least partly to the new technologies have increased their output.
(B) Less fossil fuel was used to manufacture the machinery employed in the new technologies than was originally used to manufacture the machinery employed in the older technologies.
(C) More electricity is used by those industries that have switched at least partly to the new technologies than by those industries that have not switched.
(D) Some of the industries that have switched at least partly to the new technologies still use primarily technologies that are powered by fossil fuels.
(E) The amount of fossil fuel used to generate the electricity needed to power the new technologies is less than the amount that would have been used to power the older technologies.

OA: E

My answer : C

But the OG does not provide a clear explanation even though they have mentioned that this is what most would expect. Even the right answer has no explanation.

‘E’ states that the amount of fossil fuels used is less in new technologies as compared to older technologies. This strengthens the argument no doubt.

But As per ‘C’ , since more electricity is used , obviously less fossil fuel is being used --> Isn't this stronger than Option ‘E’

Pls clarify . Thanks
e-GMAT Representative
Joined: 02 Nov 2011
Posts: 4346
Own Kudos [?]: 30782 [3]
Given Kudos: 635
GMAT Date: 08-19-2020
Send PM
Re: A recent government study links the high rates of respiratory ailments [#permalink]
3
Kudos
Expert Reply
rockybalboa123 wrote:
OG – 13. Q # 59

Which of the following most logically completes the passage?

A recent government study links the high rates of respiratory ailments in Groverston to airborne pollutants released by the Woodco plywood manufacturing plant there. To address the problem the government imposed strict regulations on emissions which will go into effect in four years. Although Woodco plans to cut its emissions in half two years ahead of schedule, it is unlikely that the rate of respiratory ailments will decline before the regulations go into effect, since _______.

A.the number of facilities capable of treating respiratory ailments is not likely to increase
B.reducing emissions even further than planned would necessitate decreasing production at Woodco
C. it is difficult to make accurate, long-term predictions about emissions
D.not all respiratory ailments are caused by airborne pollutants
E.three new plywood manufacturing plants are about to go into production in Groverston

OA: E

Why is D not the valid answer? There can be other factors as well which leads to respiratory ailments and these factors will only decline after the government regulations go into effect.

Pls help clarify this doubt. I was very confident of Option ‘D’. Was actually surprised to see that this is not the OA



Hi,

Quite interesting doubts! :)

I'll be posting my replies separately for each of the questions to limit the length of my posts.

Let's come back to this question:

Consider this:

Disease X is caused by 3 things: A, B and C.

Cause A leads to 10 incidences of disease X per year
Cause B leads to 20 incidences of disease X per year
Cause C leads to 30 incidences of disease X per year

Now, I tell you that measures have been taken to reduce cause A and therefore, the number of incidences of disease X should decline (since the number of incidences due to cause A should decline).

Can you now say that "No, you can't say that. There are other ways to acquire disease X."

No. You can't say that. I never said that we are going to eliminate disease X. I just said disease X should decline. This is going to be true, no matter how many other ways are there to cause Disease X. (unless of course other causes are increasing, which is not given in the passage). If now, cause A is going to cause only 5 incidences, there'll be only 55 incidences of Disease X as compared to 60 incidences previously.

I have addressed a similar doubt in the below question:

lyme-disease-is-caused-by-a-bacterium-transmitted-to-humans-129950.html#p1187429

rockybalboa123 wrote:
There can be other factors as well which leads to respiratory ailments and these factors will only decline after the government regulations go into effect.


What other factors do you think will decline only after government regulation go into effect?

The government regulation is only about cutting emissions. Nothing else. These emissions will be cut in half in the next two years by Woodco. Option E says that even if Woodco cuts its emission to half, there are other plants waiting to start which will negate all the decrease in pollution by Woodco.

Did I make sense? Let me know if you understand this.

Thanks :)
Chiranjeev
Magoosh GMAT Instructor
Joined: 28 Dec 2011
Posts: 4452
Own Kudos [?]: 28569 [2]
Given Kudos: 130
Re: A recent government study links the high rates of respiratory ailments [#permalink]
2
Kudos
Expert Reply
rockybalboa123 wrote:
OG – 13. Q # 59
Which of the following most logically completes the passage?

A recent government study links the high rates of respiratory ailments in Groverston to airborne pollutants released by the Woodco plywood manufacturing plant there. To address the problem the government imposed strict regulations on emissions which will go into effect in four years. Although Woodco plans to cut its emissions in half two years ahead of schedule, it is unlikely that the rate of respiratory ailments will decline before the regulations go into effect, since _______.

A. the number of facilities capable of treating respiratory ailments is not likely to increase
B. reducing emissions even further than planned would necessitate decreasing production at Woodco
C. it is difficult to make accurate, long-term predictions about emissions
D. not all respiratory ailments are caused by airborne pollutants
E. three new plywood manufacturing plants are about to go into production in Groverston


OA: E

Why is D not the valid answer? There can be other factors as well which leads to respiratory ailments and these factors will only decline after the government regulations go into effect.

Pls help clarify this doubt. I was very confident of Option ‘D’. Was actually surprised to see that this is not the OA

Dear Rocky Balboa,

I loved your fights with Apollo Creed. :-) I am more than happy to help, but I am not sure if I will have time for all three questions, so I will answer them separately. If I may say so, in the future, I think you would make it considerably easier on your respondents if you posted each question separately.

For this question, we are told explicitly ---- "A recent government study links the high rates of respiratory ailments in Groverston to airborne pollutants released by the Woodco plywood manufacturing plant there." In GMAT CR, we have to accept all evidence as unassailable. Here, we are told there is a direct link: airborne pollutants cause high rates of respiratory ailments. It may be other factors cause airborne ailments in low rates, but the government study seems to indicate the the high rate is due solely to the airborne pollutants. The argument is not about all factors that could cause even a 1% chance of respiratory ailments ---- rather, it's about a place with high rates that are unlikely to decline. Therefore, the other factors are irrelevant. That's why (D), though tempting, is wrong.

By contrast, (E) says --- three more plywood plant ---- even if Woodco & the other three each produce half of Woodco's initial rates of emission, that's still twice as much emission as when just Woodco was in town. More airborne pollutants -----> the government's study makes clear, these will cause more respiratory ailments. That's why (E) is the answer.

Mike :-)
Magoosh GMAT Instructor
Joined: 28 Dec 2011
Posts: 4452
Own Kudos [?]: 28569 [0]
Given Kudos: 130
Re: A recent government study links the high rates of respiratory ailments [#permalink]
Expert Reply
rockybalboa123 wrote:
OG – 13 . Q # 67
Several industries have recently switched at least partly from older technologies powered by fossil fuels to new technologies powered by electricity. It is thus evident that less fossil fuel is being used as a result of the operations of these industries than would have been used if these industries had retained their older technologies.

Which of the following, if true, most strengthens the argument above?

(A) Many of the industries that have switched at least partly to the new technologies have increased their output.
(B) Less fossil fuel was used to manufacture the machinery employed in the new technologies than was originally used to manufacture the machinery employed in the older technologies.
(C) More electricity is used by those industries that have switched at least partly to the new technologies than by those industries that have not switched.
(D) Some of the industries that have switched at least partly to the new technologies still use primarily technologies that are powered by fossil fuels.
(E) The amount of fossil fuel used to generate the electricity needed to power the new technologies is less than the amount that would have been used to power the older technologies.


OA: E
My answer : C

Dear Rocky
I'm happy to help with this. :-)

This is one of these questions where it's imperative to have some rudimentary real world knowledge. Here, the real world knowledge one has to have is --- where does electricity come from? It's a sad fact that, in industrial & post-industrial societies, most electricity is generated by burning fossil fuels. I would love to see more electricity generated by clean sources, like solar power and wind power, but at the moment, these clean source account for shocking small percentages of total electrical generation.

Thus, in this question, using more electricity at the factory means, yes, at the factory, more electricity and less fossil fuels, but in order to know overall whether less in fossil fuels is being consumed, we have to know --- how much fossil fuel is used to make the electricity that they will use at the factory? The argument is not about using less fossil fuels only at the factory --- it's about using less fossil fuels overall.

Here's a blog series that gives some basic real-world background in an assortment of fields that you may well have to know to understand CR questions.

https://magoosh.com/gmat/2012/gmat-supply-and-demand/

I would say, it's crucial to read the newspaper and read news journals like the Economist magazine, to have a good sense of real world contingencies. The GMAT CR always reflect real-world priorities, and they are ripe for misinterpretation if you are not familiar with these priorities.

Does all this make sense?
Mike :-)
e-GMAT Representative
Joined: 02 Nov 2011
Posts: 4346
Own Kudos [?]: 30782 [0]
Given Kudos: 635
GMAT Date: 08-19-2020
Send PM
Re: A recent government study links the high rates of respiratory ailments [#permalink]
Expert Reply
rockybalboa123 wrote:

OG – 13, Q # 65


Which of the following logically completes the passage?

Pecan growers get a high price for their crop when pecans are comparatively scarce, but the prices drops sharply when pecans are abundant.
Thus, in high yield years, growers often hold back part of their crop in refrigerated warehouses for one or two years, hoping for higher prices in future.This year's pecan crop was the smallest in five years. It is nonetheless quite possible that a portion of this year's crop will be held back,since

A) Each of the last two years produced record breaking pecan yields
B) The Quality of this year's pecan crop is no more worse than the quality of the pecan crops of the previous five years
C) pecan prices have not been to sharp fluctuation in recent years
D) For some pecan growers , this year's crop was no smaller than last year's.
E) the practice of olding back part one year's crop had not yet become widespread the last time the pecan crop was as small as it was this year.

OA:A

Why is ‘E’ wrong?

Option E states that the practice of holding back the crops had not become widespread which means that the crops were sold which means that the pecan growers had no inventory.

Furthermore, the quantity was the same. Hence they will hold back the crops this year in order to gain more profits in future



I like the way you explain your thinking. This helps me greatly in responding directly to your doubts. Thanks :)


E) the practice of holding back part one year's crop had not yet become widespread the last time the pecan crop was as small as it was this year.

Read the underlined part. When did this last time happen? Not in the last five years because it is clearly stated in the passage that this years crop was the lowest in five years. So, how can we say that pecan growers don't have inventory right now? If last year's crop was very high, then they should have the inventory.

In addition, even if they had no inventory, why would they hold back crops when the harvest is already low? It is clearly given in the passage that growers hold back the crop when the harvest is high. In case of low production, pecan growers get high prices and thus, would want to sell their whole produce. (Storing for next year would be risk since there is a chance that next year produce may be higher and prices lower)

Does that make sense?

Now, let me explain why option A is correct? If each of last two years had record harvests, then growers should have some inventory from last two years ( since they would have held back some of their produce in the last two years). In such a scenario, they would want to replace their old products in the inventory with the new produce of this year, to avoid rotting of the crops. Thus, they would hold back some of their current year produce.

Hope this helps :)

Let me know if it doesn't serve your purpose.

Thanks,
Chiranjeev
Magoosh GMAT Instructor
Joined: 28 Dec 2011
Posts: 4452
Own Kudos [?]: 28569 [0]
Given Kudos: 130
Re: A recent government study links the high rates of respiratory ailments [#permalink]
Expert Reply
rockybalboa123 wrote:

OG – 13, Q # 65

Which of the following logically completes the passage?

Pecan growers get a high price for their crop when pecans are comparatively scarce, but the prices drops sharply when pecans are abundant.
Thus, in high yield years, growers often hold back part of their crop in refrigerated warehouses for one or two years, hoping for higher prices in future.This year's pecan crop was the smallest in five years. It is nonetheless quite possible that a portion of this year's crop will be held back, since

A) Each of the last two years produced record breaking pecan yields
B) The Quality of this year's pecan crop is no more worse than the quality of the pecan crops of the previous five years
C) pecan prices have not been to sharp fluctuation in recent years
D) For some pecan growers , this year's crop was no smaller than last year's.
E) the practice of holding back part one year's crop had not yet become widespread the last time the pecan crop was as small as it was this year.


OA:A

Why is ‘E’ wrong?

Option E states that the practice of holding back the crops had not become widespread which means that the crops were sold which means that the pecan growers had no inventory. Furthermore, the quantity was the same. Hence they will hold back the crops this year in order to gain more profits in future

OK, Champ, it looks like I will have time to answer all three after all. :-)

(A) each of the last two years produced record breaking pecan yields
This means everyone has a bumper crop in reserve, so even though yields are low this year, there's going to be no shortage of pecans, as folks will be releasing frozen surplus from previous years. It makes sense that a farmer might prefer to sell some stuff frozen two years ago, and put into the freezer fresh stuff grown now, as opposed to selling the fresh stuff right now and holding two-year old stuff in the freezer for another year. Freezing helps keeps food longer, but you can't keep things forever in a freezer. This is an excellent answer.

(E) the practice of holding back part one year's crop had not yet become widespread the last time the pecan crop was as small as it was this year.
We are told in the prompt, this practice of holding back part of one's pecan crop is now standard practice --- we don't know how long it's been standard practice, but it is currently standard practice.
When was the last time pecan crops were these small? We don't know. Maybe it was 4 or 5 years ago. Or maybe it was in 1893. We don't know. If the last time pecan crops were low was 4 or 5 years ago, then what they did 4 or 5 years ago might have some bearing on what they would do now. If the last time pecan crops were low was 1893, before agribusiness, before electrical refrigeration, then whatever they did back then, however they dealt with the situation then, is completely irrelevant to how we would deal with the situation now. Because we have absolutely no idea when this "last time" was, we cannot draw any definitive conclusion from this statement.

This is why (A) is a much stronger answer than (E).

Here's a free practice GMAT CR question:
https://gmat.magoosh.com/questions/3118
When you submit your answer, the following page will have a complete video explanation. Enjoy!

Mike :-)
User avatar
Intern
Intern
Joined: 15 Jul 2009
Posts: 14
Own Kudos [?]: 45 [0]
Given Kudos: 0
Send PM
Re: A recent government study links the high rates of respiratory ailments [#permalink]
Holy Molly

Between d and e I chose d.
E. The premise clearly says that the airborne pollutants by the woodco plant. The new plants are not specified by woodco and study clearly links airborne pollutants of woodco plant and no claim abt any other manufacturing plants. Regulation just says emissions. I cannot conclude the new plants or any plant other than woodco releases harmful pollutants. Even after the regulations go into affect, it was a stretch to assume all new plants will also act like the harmful woodco plant.

Therefore I gave preference to d because irrespective of what woodco does, the world is going to be a hell with respiratory ailments.

But seems like you all have a different reasoning....
e-GMAT Representative
Joined: 02 Nov 2011
Posts: 4346
Own Kudos [?]: 30782 [1]
Given Kudos: 635
GMAT Date: 08-19-2020
Send PM
Re: A recent government study links the high rates of respiratory ailments [#permalink]
1
Kudos
Expert Reply
aagar2003 wrote:
Holy Molly

Between d and e I chose d.
E. The premise clearly says that the airborne pollutants by the woodco plant. The new plants are not specified by woodco and study clearly links airborne pollutants of woodco plant and no claim abt any other manufacturing plants. Regulation just says emissions. I cannot conclude the new plants or any plant other than woodco releases harmful pollutants. Even after the regulations go into affect, it was a stretch to assume all new plants will also act like the harmful woodco plant.

Therefore I gave preference to d because irrespective of what woodco does, the world is going to be a hell with respiratory ailments.

But seems like you all have a different reasoning....

Hi,

Even though the world may remain hell with respiratory ailments, it can become slightly lesser hell if you take one hell-creating-reason out. Therefore, we cannot say that intensity of "hell" will not decrease ( this means that option D doesn't support the conclusion).

For example: If Joe gets slaps from 100 people a day, including me. Then, if I stop slapping him, definitely his pain will reduce, even though there are still 99 people who are going to slap him.

Just my two cents. I am not sure if it helps.

Thanks,
Chiranjeev
User avatar
Intern
Intern
Joined: 28 Dec 2013
Posts: 23
Own Kudos [?]: 71 [0]
Given Kudos: 18
Location: United States
GPA: 3
WE:Information Technology (Insurance)
Send PM
Re: A recent government study links the high rates of respiratory ailments [#permalink]
If E is indeed the answer are we to assume that the 3 new plywood plants are gonna be set up without emission control ?

And why not D? Over the years the other factors that cause resp ailments may increase and sustain the number of respiratory ailment cases in Groverstone ryt?

It seems as if we are making convenient assumptions to get to the answer. Someone plz help clarify :(
User avatar
Manhattan Prep Instructor
Joined: 30 Apr 2012
Posts: 782
Own Kudos [?]: 2583 [5]
Given Kudos: 5
Send PM
Re: A recent government study links the high rates of respiratory ailments [#permalink]
2
Kudos
3
Bookmarks
Expert Reply
janxavier wrote:
If E is indeed the answer are we to assume that the 3 new plywood plants are gonna be set up without emission control ?

And why not D? Over the years the other factors that cause resp ailments may increase and sustain the number of respiratory ailment cases in Groverstone ryt?

It seems as if we are making convenient assumptions to get to the answer. Someone plz help clarify :(


In considering the impact on the 3 new plants we can only pull from the information in the argument, stating that these plants do create airborne emissions and the governmental regulations don't go into effect for 4 years. The current site will see emissions cut in half, but that is still several years away. The new plants are set to open in the near future, so even if we assume the new plants come online at 50% of the current plant's emissions we are still increasing the airborne pollution by 150%. That would certainly lead us to believe that respiratory illnesses will not decrease (answer E).

Answer choice D is a bit tempting, but one of the premises is that the "high" rates of respiratory illnesses are connected to the plants. We don't have information on the other factors, but the premises make clear that the major cause of high rates that the government is trying to reduce comes from emissions from the plant. We need an answer choice that helps us understand how we can reduce emissions at that plant and still have these elevated rates. Only choice E gives us that information.

KW
avatar
Intern
Intern
Joined: 07 Nov 2014
Posts: 2
Own Kudos [?]: 1 [1]
Given Kudos: 0
Send PM
Re: A recent government study links the high rates of respiratory ailments [#permalink]
1
Kudos
I red attentively all the answers and, as a few other questions I am training on, I have problems to understand the assumptions that are made in the correct answers (for some of them).
What if respiration ailments are caused ONLY by Woodco plant... Answer E doesn't say that the three plants will be built by Woodco, it only says that they will be plywood plants. What if the three new plants will be built by another company that has a process that they don't produce these resp ailments ?
We are here making the assumption that it's the plywood that creates the resp ailments, aren't we ? But the arguments says "government study links the high rates of respiratory ailments in Groverston to airborne pollutants
released by the Woodco plywood manufacturing plant there"

I answered D making the assumption that the government's study might be not complete and it might be that WoodCo uses a product in their process that generates airborne pollutants.

Now it's my assumption against other's assumptions. So in that case we can make many "wrong" classified answers "right" by making assumptions and we could debate many of the questions' right answers.

If I am wrong on making an assumption on a statement from the argument, and the correct answer made an assumption for one answer, then I should be free to make any assumption to make any answer correct; or that means that the corrector is free to make his own assumptions and instead of studying for the GMAT, I'd rather contact them now, and spend some holiday with them to determine their personality so I can guess what are the assumptions. :)

How about this one ?
In the last decade there has been a significant decrease in coffee consumption. During this same time, there
has been increasing publicity about the adverse long-term effects on health of the caffeine in coffee.
Therefore, the decrease in coffee consumption must have been caused by consumers’ awareness of the
harmful effects of caffeine.
Which of the following, if true, most seriously calls into question the explanation above?
(A) On average, people consume 30 percent less coffee today than they did 10 years ago.
(B) Heavy coffee drinkers may have mild withdrawal symptoms, such as headaches, for a day or so after
significantly decreasing their coffee consumption.
(C) Sales of specialty types of coffee have held steady as sales of regular brands have declined.
(D) The consumption of fruit juices and caffeine-free herbal teas has increased over the past decade.
(E) Coffee prices increased steadily in the past decade because of unusually severe frosts in coffee-growing
nations.


The correct answer is E... but I did not make the assumption that the customer has a fix budget on buying coffee and therefore stopped buying increasing price coffee (but the corrector made this assumption)... I made the one that, instead of consuming coffee, people switched to fruit juices and caffeine-free herbal teas. You see, it's my assumption against the one of the corrector.

Can someone please tell me how and when I can clearly make assumptions.
User avatar
Manhattan Prep Instructor
Joined: 30 Apr 2012
Posts: 782
Own Kudos [?]: 2583 [0]
Given Kudos: 5
Send PM
Re: A recent government study links the high rates of respiratory ailments [#permalink]
Expert Reply
lfcfan wrote:
I red attentively all the answers and, as a few other questions I am training on, I have problems to understand the assumptions that are made in the correct answers (for some of them).
What if respiration ailments are caused ONLY by Woodco plant... Answer E doesn't say that the three plants will be built by Woodco, it only says that they will be plywood plants. What if the three new plants will be built by another company that has a process that they don't produce these resp ailments ?
We are here making the assumption that it's the plywood that creates the resp ailments, aren't we ? But the arguments says "government study links the high rates of respiratory ailments in Groverston to airborne pollutants
released by the Woodco plywood manufacturing plant there"

I answered D making the assumption that the government's study might be not complete and it might be that WoodCo uses a product in their process that generates airborne pollutants.

Now it's my assumption against other's assumptions. So in that case we can make many "wrong" classified answers "right" by making assumptions and we could debate many of the questions' right answers.

If I am wrong on making an assumption on a statement from the argument, and the correct answer made an assumption for one answer, then I should be free to make any assumption to make any answer correct; or that means that the corrector is free to make his own assumptions and instead of studying for the GMAT, I'd rather contact them now, and spend some holiday with them to determine their personality so I can guess what are the assumptions. :)


First, let me shift your mindset. Your job in critical reasoning is not to make assumptions. You job is to determine which assumptions are made by the argument.

Take this argument about respiratory illnesses. The premises state that illness is connected to the Woodco plants. Woodco plans to reduce emissions but that won't reduce the amount of illness. What could cause that? It seems that we are addressing the issue of Woodco emissions, so why won't that change reduce the amount of illnesses? We need to find the answer that provides additional information that helps us understand why the plan won't work. Only E gives us information that would explain why the reduction in emissions won't reduce illness - more emissions are going to come online. The assumption that this statement attacks is that in the process of fixing the Woodco emissions you don't add more emissions to the environment. We didn't make this assumption - this was an assumption that was part of the original argument.

KW
User avatar
Manhattan Prep Instructor
Joined: 30 Apr 2012
Posts: 782
Own Kudos [?]: 2583 [0]
Given Kudos: 5
Send PM
Re: A recent government study links the high rates of respiratory ailments [#permalink]
Expert Reply
lfcfan wrote:

How about this one ?
In the last decade there has been a significant decrease in coffee consumption. During this same time, there
has been increasing publicity about the adverse long-term effects on health of the caffeine in coffee.
Therefore, the decrease in coffee consumption must have been caused by consumers’ awareness of the
harmful effects of caffeine.
Which of the following, if true, most seriously calls into question the explanation above?
(A) On average, people consume 30 percent less coffee today than they did 10 years ago.
(B) Heavy coffee drinkers may have mild withdrawal symptoms, such as headaches, for a day or so after
significantly decreasing their coffee consumption.
(C) Sales of specialty types of coffee have held steady as sales of regular brands have declined.
(D) The consumption of fruit juices and caffeine-free herbal teas has increased over the past decade.
(E) Coffee prices increased steadily in the past decade because of unusually severe frosts in coffee-growing
nations.


The correct answer is E... but I did not make the assumption that the customer has a fix budget on buying coffee and therefore stopped buying increasing price coffee (but the corrector made this assumption)... I made the one that, instead of consuming coffee, people switched to fruit juices and caffeine-free herbal teas. You see, it's my assumption against the one of the corrector.

Can someone please tell me how and when I can clearly make assumptions.


So, just like before, you need to remember that you are not going to make assumptions. You have to identify which assumptions are made by the argument. The assumptions sit in the logical gaps between premises and the conclusion. They are the unwritten elements that have to be true for the assumption to be valid.

Here the argument states that coffee consumption has decreases over the same period that publicity has discussed the health impacts of caffeine. The conclusion states that the publicity is the cause of the decrease. Your job is to attack an assumption made between the premises and conclusion. You could probably think up some assumptions on your own, but this doesn't make sense on a GMAT question because assumptions are basically limitless and it's hard to predict which assumption the GMAT will be using on any given problem. You need to recognize the assumptions (or in this case the attacks on assumptions) as you go through the answer choices.

A-This supports the premises
B-This is out of scope
C-This one is tempting because it talks about a decline but doesn't offer an alternate explanation for that decline
D-Also tempting, but it doesn't directly explain the decrease in coffee - there isn't a direct connection to say that coffee drinkers have switched to juices and teas.
E-This provides us with an alternate explanation for the decrease - people aren't willing to pay more money for their coffee. This attacks the assumptions that people are not price sensitive about their coffee buying (see - you never would have guessed that when you first read the argument)

KW
avatar
Intern
Intern
Joined: 07 Nov 2014
Posts: 2
Own Kudos [?]: 1 [0]
Given Kudos: 0
Send PM
Re: A recent government study links the high rates of respiratory ailments [#permalink]
Kyle, thank you very much for your answers.
Just for me to understand, on the latter,
Answer E makes clearly the assumption that the drop of coffee consumption is caused by the increase of coffee price. In answer D, why is the assumption that the coffee consumption drop is caused by people switching to herbal tea and juice wrong ?
You say that "there isn't a direct connection to say that coffee drinkers have switched to juices and teas" and I agree. But there is none either that says that people are not paying for the price difference coming from the steady increase of coffee price.
GMAT Club Bot
Re: A recent government study links the high rates of respiratory ailments [#permalink]
 1   2   
Moderators:
GMAT Club Verbal Expert
6920 posts
GMAT Club Verbal Expert
238 posts
CR Forum Moderator
832 posts

Powered by phpBB © phpBB Group | Emoji artwork provided by EmojiOne